LSAT PT 14 Expl Unlocked

You might also like

Download as pdf or txt
Download as pdf or txt
You are on page 1of 36

LSAT

PrepTest 14
Explained
a guide to the february, 1995 exam
© 2006 Kaplan, Inc.

All rights reserved. No part of this book may be reproduced in any form, by photostat, microfilm, xerography, or any other
means, or incorporated into any information retrieval system, electronic or mechanical, without the written permission of
Kaplan, Inc. LSAT is a registered trademark of the Law School Admission Council.
Section I: Logic Games

SECTION I: These basic rules are just as important as the indented


ones. If you missed them, the game is undoable.
LOGIC GAMES
1) Only one president means that only one of our
employees will go unsupervised. Note how Logic
Game 1: Company Employees Games tend to reflect real life situations. It makes
sense that there is only one president, just as it makes
Questions 1–6 sense that the employees can only supervise those
that are of a lower level.
The Action: We’re asked to group five employees—F, 2) At least one of the people that the president
G, H, K, and L—(a Godsend; more entities would have supervises will be a manager. Don’t make the mistake
made this game considerably less manageable) into of assuming that this rule means that the president
three categories—president, manager, and technician. must supervise more than one employee; that’s not
There’s an extra dimension in this one: Some what the rule states. It’s quite possible that the
employees supervise other employees. The Key Issues president super vises exactly one employee, a
are: manager. At this point though, the president could also
1) What employees can, must, or cannot be distributed supervise one or more technicians.
into each group? 3) This rule will prove vital. We know that the president
2) What employees can, must, or cannot supervise is unsupervised (opening paragraph), and that a
what other employees? manager cannot supervise another manager (opening
paragraph). So if each manager supervises at least
The Initial Setup: This is not an easy setup to get a one employee, we now know that there must be at
handle on, because we want to include not only the least as many technicians as managers.
consideration of position, but also who supervises
whom. One way is to mimic a “family tree” type of 4) Don’t just write “F supervises no one.” Consider
game, with entities placed above and below each other what that means. We learned from the setup that only
and connected by lines. This way we can place entities technicians do not supervise anyone, so instead of
who supervise over those they supervise and connect rewriting the rule, we can put F directly into our diagram
them with lines, like so (the exact numbers of each as one of the technicians.
position will have to wait for the rules): 5) You will want to make a note reminding yourself that
G must supervise two employees, but once again, first
stop and think about what this tells us. We can put G
President right into our diagram by noting that she must either be
the president or at least a manager (since technicians
FGHKL Manager do not supervise anyone).

Technician Key Deductions: We’ve already jumped the gun and


made a few deductions from the opening paragraph
and the rules (it’s okay to deduce as you go along; we
The Rules: usually discuss deduction in a separate paragraph in
This game has many vital rules built into the opening these written explanations for the sake of clarity. But
paragraph: there’s more to think about before moving on to the
We’re told that only the president is not supervised, questions. With only five entities and one (F) solidly
and that each employee is supervised by one employee fixed to a group (Technician), you can bet that it would
who is either the president or a manager. We can infer be worth our time to consider the numbers game and
then that technicians cannot supervise anyone. drive it to something concrete. Five entities and exactly
one of them is president. What could be the numbers
We’re also told that the supervised employee has a breakdown of president, managers, and technicians,
different position than his or her supervisor. Since the respectively? The only possibilities are 1, 1, 3 and 1,
supervised employee and the supervisor cannot hold 2, 2; we inferred from Rule 3 that the number of
the same position, this means that all managers are managers can’t be greater than the number of
supervised by the president and all technicians are technicians.
supervised by a manager or the president.

1
PrepTest 14 Explained

We could even take this thinking one step further by The Questions:
combining it with Rule 5. If G is a manager, then the
setup has to be 1, 1, 3. Do you see why? If G’s a 1. (B)
manager, the requirement that she supervises exactly We love these acceptability questions. Just check each
two employees (technicians in this case), wouldn’t rule against each choice. Rule 1 downs (D). Rule 3
leave any technicians for the other manager to cuts (A) and (E). Rule 4 (among others) takes care of
supervise in the 1, 2, 2 setup. (C), which leaves us with our answer.
Going another step further, what if we know that there
are two managers? Well, that means that the setup is 2. (A)
1, 2, 2, and G must be the president. G can only be Here’s where they test our deduction. This we already
the president or a manager, and we just showed that G know: exactly 1 president (Rule 1), at least 1 manager
can’t be a manager unless she’s the only one. (Rule 2), so a maximum of 3 technicians.
This is quite a bit of deducing to do up front; don’t get
the idea that you couldn’t have handled this game 3. (E)
without working all of this out up front. But if you’re We can cross off (A) and (B), thanks to our first
able to deduce this far, it saves you a tremendous deduction that F is a technician. We also deduced that
amount of time in the long run. if there are two managers, then the president must be
G. Since G can’t be one of two managers, we can cross
The Final Visualization: Here’s what we’re armed with off (C) and (D), leaving (E). If you hadn’t made that
as we move on to the questions: second deduction, no problem; you’re simply forced to
make it now: If G is one of two managers, then there
1/2/2 or 1/1/3 needs to be three technicians: Two for G to supervise
G President (Rule 5) and one for the other manager to supervise.
G This is no good, because it leaves no one to be
G supervises 2 Manager president.
HKL
4. (B)
Technician
To the choices:
(A) No way. From the beginning we’ve said that there
F F has to be at least two technicians.
The Big Picture: (B) Yes. We also deduced at the start that this is indeed
possible (as long as G is our president). On test day,
• This section is a perfect example of why Kaplan mark (B) and go on, but to get some practice at
suggests taking time to do an overview of the spotting wrong answer choices, we’ll continue.
section before plunging in. This game is one of
(C) No; the only employee who isn’t supervised is the
the trickiest in the section. You would have been
president, and there’s only one president, so only one
better served if you left it towards the end, after
employee will go unsupervised.
you’ve already racked up some points (and
confidence) with the other games. (D) No, we know that the only two possibilities are 1,
1, 3 and 1, 2, 2. There can’t be more managers than
• Critical reading is just as important in Logic
technicians, thanks to Rule 3.
Games as it is in Logical Reasoning and Reading
Comp. If you just glazed over the opening (E) Rule 2 said that at least one of the employees that
paragraph, you missed the heart of this game. the president supervises is a manager. Rule 3 says
Remember: All rules are not indented. that every manager supervises at least one employee.
Since every employee is only supervised by one
• Always take number-related info as far as you
employee, (E) is not possible.
can, especially in grouping games where you’re
asked to distribute entities into various subgroups.
Sometimes they tell you how many go in each.
When they don’t, it’s up to you to at least attempt
to figure out the possibilities.

2
Section I: Logic Games

5. (D)
Start by working with the new information. F (a
technician) is supervised by the president, and we
know (Rule 2) that the president also supervises at
least one manager, so the president supervises at
least two employees. But since F is supervised by the
president, the manager whom the president is
supervising needs another technician to supervise
(Rule 3). So now we have one president, one manager,
and two technicians with only one more employee to
place. Learn to ask yourself the relevant questions.
Could it be a manager? No, all of the technicians are
already being supervised, which would not leave
anyone for a new manager to supervise. We therefore
must be dealing with the 1, 1, 3 setup, which gives us
our answer.
(A) No, G could indeed be that one manager supervising
two of the three technicians.
(B) No, G could be the president. H could either be the
one manager or even one of the technicians.
(C) No, L could be the president or the manager.
(E) No; as we’ve figured out, there are exactly three
technicians.

6. (C)
So now K as well as G both supervise two employees.
Since the maximum number of technicians (as we
deduced) is three, either K or G must be president
while the other is a manager (if they were both
managers, they’d need four technicians to supervise).
Where to now? Well, one of the two people supervised
by the president can be K or G (whoever isn’t the
president), but whoever isn’t president is going to need
two technicians to supervise. The president is also
going to need another employee to supervise. Can it be
a manager? No, because, just as in question 5, there
is only one employee left, and if that person was a
manager, there would be no one left for him or her to
super vise. So K and G must be president and
manager, in either order, and all of the other
employees, including L, must be super vised
technicians.
(A) only could be true. F could be supervised by either
G or K.
(B) only could be true. G could just as easily be president.
(D) is wrong. We deduced that there can be only one
manager (G or K).
(E) Actually, we figured out that there are (once again)
exactly three technicians.

3
PrepTest 14 Explained

Game 2: Washing Dishes 4) Another loophole closer, and again don’t give it a
second thought. No two objects are washed at the
Questions 7–12 same time (no “Octopus Ron” doing the dishes).

The Action: Here we have a sequencing game within a Key Deductions/Final Visualization: We can further
sequencing game. Three types of dishes, seven flesh out our work with Rules 2 and 3. Thanks to these
objects in all, are being washed: China (a mug and a two rules, there are only two possible options for the
plate), glassware (a water glass and a juice glass), and order of dishes washed:
utensils (a fork, a knife, and a spoon). Here in the
opening paragraph we get another important rule: The Option 1 1 2 3 4 567
objects within each type are washed consecutively. K = 5 or 6 M P (W) (J) K … S
This means, for example, that the fork (a utensil) will S = 6 or 7 (J) (W) (F)
not be washed in between the mug and plate (pieces F = 5, 6 or 7
of china). However, within each group, the order in or
which the objects are washed can var y, thus
accounting for the sequence within a sequence. There Option 2 1 23 4 5 6 7
are only two basic sequencing Key Issues in this one: K = 1 or 2 K … S (W) (J) M P
1) Which objects are washed in which order? S = 2 or 3 (F) (J) (W)
2) Which objects are washed before and after which F = 1, 2, or 3
other objects?
And that’s it. From this we can see many valuable facts
The Initial Setup: This begs for a simple setup. For now (M can only be washed first or sixth, etc.) These two
just separate the dishes into type and wait for the possibilities (our “Final Visualization,” really) is what
rules: we will come back to again and again to answer nearly
all of the questions. After making the abstract “C, G, U
China Glass Utensils or U, G, C” more concrete, we have an infinitely better
MP JW FKS grasp of what’s going on.

The Big Picture:


• Always attempt to make the abstract concrete.
The Rules: • Critical reading is just as important in Logic
Games as it is in Logical Reasoning and Reading
1) is your typical loophole closer. Don’t let it worry you. Comp. This section’s games all have important
All it means is that Ron won’t go back and rewash any concepts buried in the opening paragraphs.
of the dishes. Once is enough.
• Washing dishes is something we all can relate to
2) This rule has tricky wording, but once you comprehend (unfortunately), and therefore this was probably
it, it’s quite helpful. The glasses are washed after the easier to get a handle on than the first or third
china or after the utensils, but not after both. Again, games. This is yet another argument for quickly
make the information concrete. What are the looking at all of the games in a section before
possibilities? The “not after both” part eliminates C, U, deciding where to start. Take command of the
G and U, C, G, so the “type” order will either be C, G, LSAT; don’t let the testmakers dictate how you
U or U, G, C. In other words, the two glasses, W and J, proceed.
will always be washed in the middle of the three types.
3) deals with the internal order of the dish washing.
There are only two pieces in the china category, and we
now know that the mug is always washed before the
plate. Likewise we are told that of the utensils, the
knife is washed before the spoon. Note though, that
the fork can float around anywhere in relation to the
knife and spoon. Write “M…P” and “K≥S” as reminders.

4
Section I: Logic Games

The Questions:

7. (E)
By scanning the two possibilities, it’s pretty easy to
see that the plate can only be washed second or
seventh, not third.
(A) through (D) all could be true.

8. (A)
Again, using what we know about the two possibilities,
the second option shows that the fork could be washed
first, in which case the knife would be washed second.
(B) through (E) are all impossible, as we see with our
ordering.

9. (B)
Simply check each choice against the possibilities:
Knife, fork, juice glass? For 2, 3, and 4? That’s way off.
The only order close (as we see with our two
possibilities) would be knife, fork, spoon in the 1, 2, 3
slots.
(A), (C), (D), and (E) are all very possible.

10. (C)
Again, use our two possibilities. The knife can’t be
washed third. First, second, fifth, sixth—(A), (B), (D),
(E)—yes, but not third.

11. (E)
Ask yourself, “what does this mean in the context of
this game?” The only way for this to be possible is if
the utensil order is knife, spoon, fork. Now test out the
options using this new restriction. (A) and (B) would
relate to the second option, but are both impossible
because they violate the K, S, F order. The plate can
never be washed third or sixth, in either option, so (C)
and (D) are out, but the plate can be washed seventh
if K, S, and F are washed first, second, and third,
respectively.

12. (A)
Ask yourself, “how can this be possible?” Only if the
order by type is C, G, U, our first option, and the knife
is washed fifth. The plate is washed second, and the
fork either sixth or seventh, so (A) is obviously false.
(B) and (E) could be true, and (C) and (D) must be
true.

5
PrepTest 14 Explained

Game 3: Birds on Display So make a note that one of the exhibited pairs will
always be a pair of parakeets. We can make this even
Questions 13–18 more concrete by writing “T or W” for the female space
of one of the exhibited pairs. By extension, if J or W is
The Action: This may have been the hardest game of exhibited, then S can’t be, which means that Q or R
the bunch, simply due to the sheer number of entities must be exhibited (if not, we wouldn’t have enough
(10), and the number of details involved in the game’s cages to split up our three bad-tempered male
action. And once again, there’s lots of important parakeets.) It should be obvious by now that emphasis
information found in the opening paragraph. At the should be placed on the three male parakeets.
heart of it, this game is really another grouping game
of distribution: Ten birds—H, J, K, M, N, Q, R, S, T, and The Final Visualization: There’s still a lot left open, but
W—will be placed into either an exhibition, cage 1, or this will aid us in pursuing the answers:
cage 2. Here’s where it gets more complex: Each bird
If J or W —>No S
is one of three different types—goldfinch, lovebird, and
Cages Exhibition If S —>No J, No W
parakeet—and each is either male or female (no
surprise there, but it does add another element to keep T or W 1 Pair Parakeets
track of). Up to two pairs, consisting of a male and a
female of a single type (for example, parakeets S and ( )
T), will be exhibited, and the rest will be placed into two Same sex, kind —>Diff
cages. The Key Issues are: Cages
1) What birds can, must, or cannot be exhibited with
what other birds? The Big Picture:
2) What birds can, must, or cannot be put into a cage • Whenever you’re given a sketch, use it. We
with what other birds? couldn’t have divided the birds into neater groups
The Initial Setup: The roster of birds is already if we tried.
helpfully included right on the page. And like the game • While there wasn’t really a true “process” game
on page 219 of Unit 4B in your lesson book, when in this section, this game has one particular
you’re given a diagram, use it. You could picture the aspect characteristic of “process” games—the
groups that we’ll put the birds in like this: sketch is less important, while returning to the
rules will probably come in handy in most of the
Cages Exhibition questions.
1 Pair Definite • Sometimes, a quick glance at the questions can
help you pick up on the action of a game. Here,
( ) Maybe 2 question 13 nicely lays out the grouping paradigm
in a visual way.
The Rules:
1) Each cage has a limit of four birds. “4 Max” should The Questions:
suffice.
13. (D)
2) Birds of the same type and sex cannot be put in the
An acceptability question, so check each rule against
same cage. (For example, J and K, the two female
the choice: The opening paragraph (pairs of birds are
goldfinches, will have to be split up if neither of them
exhibited), as well as Rule 1, kills choice (E). Rule 2
is exhibited.)
axes choices (A) and (C). Rule 3 eliminates (B), and
3) If either J or W is exhibited, then S can’t be. Don’t just like that we’re left with (D).
neglect the contrapositive: If S is exhibited, then J and
W are not. 14. (D)
Key Deductions: Rule 2 is very helpful—it allows us to We deduced that at least one of the male parakeets
deduce that at least one of the three male parakeets must always be exhibited (thanks to our work mainly
must always be exhibited (with a female, of course). If with Rule 2), so only consider those choices that
not, with three of them and only two cages, we’d include Q, R, or S which are (B), (D), and (E). (B) and
inevitably have to place two of them in the same cage. (E) both violate Rule 3, so only the pair in (D) is possible.

6
Section I: Logic Games

15. (D)
And here they are again. Two of the three male
parakeets are caged, so the third, S, must be
exhibited. The contrapositive of Rule 3 tells us that J
and W can’t be exhibited and must be caged. That’s
quite a bit of good work, so stop and scan the choices.
There it is, choice (D).
(A), (B), and (C) could be true. (E), no way.

16. (D)
One of our three male parakeets, Q, is assigned to the
cages, which means that one of the remaining two, R
or S, must be exhibited. And since one of the only two
female parakeets is also caged, R or S must be
exhibited with the only remaining female, W. The mere
sight of W should lead you straight to Rule 3—W can’t
be exhibited with S, so the pair of parakeets that must
be exhibited is R and W.
(A), (B), (C) could be true. (E) is always possible.

17. (B)
Not much to do but try out the choices, keeping our
three male parakeets in mind, of course.
(A) The correct answer for acceptability question 13
allows us to axe this choice.
(B) Boy, do they know how to test a concept, or do they
know how to test a concept? You’ll probably here this
in your sleep by now: “One of the pairs of exhibited
birds must be parakeets.” This is our answer.
(C), (D), and (E) all have the required parakeet pair.
You can try them out if you care to. Do note that for (E)
to be possible, Rule 3 tells us that S can’t be in either
of the two pairs.

18. (E)
Rule 3 forces T to be the female parakeet exhibited
with S, and also forces W and J into cages. No help on
its own, but that uses up all of the female parakeets.
So the other two males, Q and R, have to go in
(separate) cages. Scan the choices—yup, R in a cage.
(A) through (D) only could be true only.

7
PrepTest 14 Explained

Game 4: Seasonal Sports Key Deductions: Like good LSAT test-takers, we


considered the implications of Rule 1 with each
Questions 19–24 successive rule as we went along. Now let’s go back
and do the same with Rule 6. Otto plays hockey in the
The Action: A straightforward matching game. We’re winter (we deduced this by combining Rules 1, 3, and
asked to match up two kids, Nikki and Otto, with the 7). Since Nikki therefore can’t play hockey in the winter,
sport—hockey, kayaking, mountaineering, running, or Nikki must play volleyball in the winter (which, by the
volleyball—that they play in each of the four seasons. way, turns question 19 into a nice ‘gimme’). Now that
Predictably, the Key Issues deal with: volleyball is taken care of for both kids, cross of all the
1) What sports can Nikki and Otto play in each season? “Vs” from the other seasons. You may also have
noticed that since Nikki doesn’t play hockey in the
2) What sports can’t Nikki and Otto play in each season?
winter (she plays volleyball), that means that Nikki
The Initial Setup: Either a grid or a list would work well never plays hockey (which, by the way, turns question
for this game. For our purposes, we’ll use a 2 X 4 grid 20 into another welcome ‘gimme’).
with Nikki and Otto on the side and the four seasons
The Final Visualization: Here’s our final sketch, which
(the real ones, not the singing group) on the top, like so:
contains a great deal of information:

F W SPR S F W SPR S
Nikki Never H Nikki M/R V K/M/R K/M All Diff
Otto Otto M/R H K/M/R V All Diff
Diff Diff Diff Diff
The Rules:
There’s something concrete way down at the bottom, The Big Picture:
so let’s start with that:
• Did you do this game first or second? If not, why
7) Otto plays volleyball in the summer. Great, build it not? Did you at least get to this one? That’s why
right into the grid. previewing a section is so important. If you never
1) Once a kid plays a sport, that sport is out for the even had the chance to get to this game, you
rest of the year. For example, since Otto plays missed many easy points. Take control. You are in
volleyball in the summer, volleyball is off limits for him charge. You decide what to answer and what to
for the fall, spring, and winter. skip. You decide the order.
2) The only sports that the kids can play in the fall are • Don’t be intimidated by a large number of rules.
mountaineering, running, and volleyball. Note again, It may look scary on the page, but actually, a
Otto plays volleyball in another season, so Rule 1 large number of rules normally makes for a much
means that his fall sport is either mountaineering or more manageable game. It’s the games with
running. Indicate these things somewhere in the grid. fewer rules that are ore difficult because they’re
3) Only hockey and volleyball may be played in the much more ambiguous and abstract.
winter. Once again, since Otto plays volleyball in the
summer, he can’t play volleyball in the winter, so Otto’s The Questions:
winter sport must be hockey. Put a huge “H” in Otto’s
winter column, and “H, V” in Nikki’s winter column. 19. (B)
4) We’re used to this by now. Put “K, M, R, V” over the One of our bid deductions: Nikki plays volleyball in the
spring column. Note that volleyball is still out for Otto. winter.
5) Otto’s summer sport is volleyball, so this rule really (A), (C), (D), and (E) all could be true only.
applies only to Nikki: Put “K, M, V” in the summer
column for Nikki. 20. (A)
6) Here’s the rule that really opens up the entire game: Our second gimme. We noticed before that indeed
Nikki and Otto will not play the same sport in any hockey can’t be played by both kids. It’s only available
season. Since this will serve to greatly restrict the in the winter, when Nikki’s busy playing volleyball. (B)
possibilities, this rule is huge. through (E) are all sports that each kid could play.

8
Section I: Logic Games

21. (C) (D) If Nikki’s spring sport is R, then her fall sport would
If Nikki’s fall sport is running, then running is out for have to be M. Rule 6 then makes it impossible that
Otto in the fall (Rule 6). Volleyball is already out for him Otto’s fall sport is M.
(he plays it in the summer), so the only thing left for (E) If Nikki’s summer sport is M, then her fall sport
Otto in the fall is mountaineering. has to be R. That would force Otto’s fall sport to be M
(A), (B), (D), and (E) all could be true, but we don’t (Rule 6 again). Rule 1 makes it impossible for Otto’s
have the information to tell for sure. spring sport to also be M.

22. (B)
Not much to do except try out the choices.
(A) No way. We know (and see quite clearly in our grid)
that Nikki must play either M or R in the fall (V is out,
since she plays it in the winter).
(B) Sure, Nikki could play K in the spring, M in the
summer, and R in the fall, along with V in the winter.
This is possible, so it’s our answer.
(C) Same as (A). Since V is out for Nikki, she’s down
to K or M in the summer.
(D) Otto also has to play M or R in the fall because he
plays V in the summer.
(E) These are exactly Otto’s options for the spring: V’s
exclusion means that Otto’s only spring choices are K,
M, or R.

23. (B)
Since Otto was down to M or R in the fall, this means
it must be M. Rule 6 therefore forces Nikki’s fall sport
to be R, cross off (A). Rule 1 eliminates R from Nikki’s
choices for the rest of the year, and there’s our answer.
She can’t run in the spring.
It turns out that every sport can be determined in this
one; (C), (D), and (E) all must be true.

24. (C)
Again, not much to do but try out each choice.
Remember that the Rule 1 makes it necessary to make
sure that the entire year works out. So we’ll just plug
in the info and see if we can come up with a year that
works for both kids.
(A) If M is Nikki’s fall sport, then R would have to be
Otto’s fall sport. It couldn’t be his spring sport, too.
(B) If Nikki’s spring sport is R and her summer sport
is M, she’s out of possibilities for the fall (we deduced
way back, that M and R were her only fall options).
(C) No problem. Here’s what we get: fall: Nikki—R,
Otto—M; winter: Nikki—V, Otto—H; spring: Nikki—M,
Otto—R or K; summer: Nikki—K, Otto—V.

9
PrepTest 14 Explained

SECTION II: 3. (B)


LOGICAL REASONING Normally deer mice don’t have a clue when it comes to
getting back to their nests. What’s different about this
case? The only clue you get is that the researchers
1. (C) camped near the deer mouse’s nest for a time, and
The paradox is: How is it possible for the city’s water that the mouse found its way back to the nest near the
shortage to get worse, even as the city receives researchers’ camp. This suggests that presence of the
unusually heavy rainfall? (C) gives the answer: People camp may have helped the mouse find its way home.
were simply using more water throughout the month of (B) explains how this could happen; the mouse could
June, which presumably counterbalanced the effect of have followed the smell or trail of the smoke back to
added rain. So people saw the rain falling, threw the location of the camp, and then found its nest.
conservation to the winds, and used so much water (A), (C), and (E) all side-step the issue by simply
that the city suffered a net water loss, despite June’s describing the new area to which the mouse had been
heavy rainfall. moved. (A) tells us the terrain is rocky; (C) that there
(A) is irrelevant; it does nothing to explain how the are few other deer mice in the area; (E) that there are
water shortage managed to get worse during the month many prey animals in the area. But nothing in any of
of June. these features relates to the ability of the mouse to
(B) Au contraire. By showing people using less water in find its way home. We’re only interested in how this
June, (B) makes the deepening water crisis harder to mouse succeeded where so many others have failed—
understand. describing the new environment doesn’t explain that.
(D) has nothing to say about the particular period we’re (D)’s description of the manner in which the mouse
interested in (the month of June), and moreover it gives was moved also isn’t helpful. In fact, its being kept in
a reason for the shortage to be not so severe. a dark box removes one possible (admittedly
(E) is a flat-out au contraire choice; if per-capita use of farfetched) explanation for how it found its way home—
water declined in June, we’d expect the water shortage that it looked out the window and retraced its steps.
to become less severe.
4. (E)
2. (C) Simple argument here: Arts groups survived an 8
There are only two ways an applicant can qualify: either percent cut in public funding last year, so they can
have an earned doctorate and a record of published survive another 8 percent cut in public funding this
research or have five years’ work experience. The only year. This should have struck you as fishy; just
fields that count toward either qualification are because the groups managed to survive one funding
sociology, psychology, and education. We’re looking for cut doesn’t mean they can sur vive further cuts
someone who fails to meet either qualification, and indefinitely. (You’ve certainly heard of the straw that
that’s Edward St. John, (C). He’s done the work, and breaks the camel’s back.) That’s exactly the flaw that
earned the doctorate and published a lot, but his work (E) points out: The “cumulative” effects of the cuts
and doctorate were in the field of business, and his might reduce the arts groups’ funding so much that
publications were novels, not research. Sorry, Mr. St. they can no longer survive.
John, no job for you. (A) The argument never ties the survival of the arts
(A) Joanne Berstein has the work experience—eleven groups to an improvement in the economy; in fact, the
years at the department of education. No need to read author says that the arts groups managed to raise
the rest of her “resume.” enough private money last year despite a recessionary
economy, which suggests that a good economy isn’t
(B) Alvin Johnson also has the work experience— necessary for the groups’ survival.
seven years in the field of psychology.
(B) is a “beyond the scope” choice. The argument is
(D) Michael Roberts’ “resume” is a little harder to follow, trying to establish the factual conclusion that arts
but his two years teaching education classes and his groups will survive another funding cut; in order to
four years work on an education task force add up to establish that conclusion, the argument needn’t
six years work in the field of education, so he’s OK. address the issue of whether there should be public
(E) Alicia Arias gets by on her academic record; she funding of the arts.
holds an earned doctorate in sociology and she’s
published plenty of sociological research.

10
Section II: Logical Reasoning

(C) Again, outside the scope—the argument never brings confirming the most important survival attribute of
in the concept of “flourishing,” or equates it with mice, their ability to survive in environments too harsh
survival. to support humans. According to (D), mice have
(D) The possibility raised by (D) actually seems to already demonstrated they can survive where human
strengthen the argument by minimizing the effect of this beings cannot—in Antarctica.
year’s cuts. In fact, however, the actual dollar amount (A) Irrelevant: This describes something that can limit
of any particular year’s cut isn’t the point; as (E) says, the mouse population (lack of food), which gives us no
the real flaw is that the argument fails to see that many reason to think mice can survive without humans
small cuts can have a large cumulative effect. (especially since mice probably get a lot of food from
humans).
5. (D) (B) So what? The reproductive ability of mice is less
Seems like another paradox: The number of books sold important to their survival than their ability to adapt to
has increased, but the time spent reading by the different environments, and we get nothing about that
average literate person has decreased. Anything that here.
helps resolve this discrepancy must somehow affect (C) ignores the claim that the ability of mice to survive
one of these factors. The best approach here is to in nature has diminished because of contact with
eliminate the choices that seem to resolve the humans; we want something that indicates that mice
seeming discrepancy; the choice that remains will be can still make it on their own, not that they once could.
the answer. (D) fails to resolve anything—how does the
(E) Au-contraire; if anything, (E) implies that most mice
fact that people used to display large book collections
live in man-made environments, which certainly doesn’t
explain why more books are sold now, or why literate
show they don’t need humans to survive.
people spend less time reading now? If anything, (D)
would lead us to expect more books to be sold fifty
7. (D)
years ago, to people trying to build up impressive
libraries. All zebras have stripes so stripes must have some use.
But they can’t work as protective camouflage for zebras
(A) explains how the total number of books sold could
that live in grasslands, says the author, so they must
have risen even though the average literate person
serve some other function. The author decides that
spends less time reading; there are simply many more
stripes must somehow serve as signals to other
literate people around now, so even if they spend less
zebras. (D) supports this with evidence that zebras
time reading on average, they could certainly buy more
respond in a special way to striped shapes: They react
books in total.
more quickly when they see striped shapes moving.
(B) attacks the idea of “number of books sold.” It gives (Possibly, this helps zebras move together and function
a valid explanation for why more books are sold now— as a herd.) At any rate, (D) supports the author’s belief
fifty years ago, readers were less likely to buy their that stripes serve as signals to other zebras by
books because they borrowed them from the library demonstrating a specific reaction that zebras have to
instead. stripes.
(C) gives us a group that might be spending the same (A) gives further evidence that stripes are good for
or even less time reading as it did in the past, yet zebras, but doesn’t support the author’s contention
would be buying many more books than in the past. that stripes function by signaling other zebras in some
(E) attacks the idea of “time spent reading.” If books way.
are shorter and easier to read than they used to be, (B) suggests that stripes may serve as camouflage in
then it’s quite possible that literate people spend less grasslands, which undermines the author’s contention
time reading on average, even if they buy more books. that stripes must have some function besides
camouflage.
6. (D)
(C) discusses a different kind of visual signal among
A new record—two mice questions on one page. The animals of the same species—color changes—but that
naturalists deny that mice depend on humans for doesn’t help establish that zebra stripes work as such
survival and predict that mice could survive even if the an intraspecies signal.
environment became too harsh to support human life.
(E) is irrelevant; zebras’ ability to whinny and snort to
Their reasons are that mice reproduce rapidly, and
each other proves nothing about the functioning of their
that, “more important to their survival,” they can adapt
stripes.
to all sorts of habitats. (D) supports their prediction by

11
PrepTest 14 Explained

8. (B) (B) is all wet; the claim isn’t an example, but is itself
The author charges that the editorial writer’s attitude a general principle, and it doesn’t concern the goals of
towards government restrictions on academic freedom Ph.D. programs—that’s discussed in the first sentence.
is inconsistent. The editorial writer first excuses U.S. (D) First, the claim doesn’t “provide evidence;” it’s a
restrictions on academic freedom, saying that conclusion based on evidence already provided.
scientists who receive public funding can’t detach Second, the argument never differentiates between
themselves from politics, then turns around and humanities and other disciplines.
condemns the Soviet government for saying what (E) gets the flow of the argument reversed; the
seems to be the same thing, that scientists can’t obser vation that the disser tation requirement is
detach themselves from politics. The author complains counterproductive is meant to support the claim that
that the editorialist hasn’t explained the difference dissertations shouldn’t be required.
between the two cases, so as things stand, it seems
that the editorialist is treating the same thing in two 10. (B)
different ways. That’s inconsistency.
Penglai merchants protest the plan to reduce outdoor
(A) The author doesn’t question the editorialist’s advertising on the grounds that it would reduce the
facts, but rather is critical of the way the editorialist overall volume of business on the island. They reason
both accepts and rejects the same principle. that a report has shown that Panglian businesses that
(C) The author doesn’t come up with an exception to a advertise do better than those that do not. But this
general claim; instead, she complains that the evidence only shows that advertising gives businesses
editorialist accepts a general claim (scientists can’t a competitive edge, not that advertising increases the
detach themselves from politics) in one case and total amount of business done on Penglai. As (B) says,
rejects it in another. the merchants assume that advertising hasn’t simply
(D) The author doesn’t refute the editorial’s basic diver ted customers from businesses that don’t
assumption, but complains that the basic assumption advertise to businesses that do advertise. If you deny
(or principle) behind the editorial seems to change. this assumption and say that advertising simply
redistributes customers, then the claim that cutting
(E) The author doesn’t deny the editorial’s conclusion
advertising will hurt overall business falls apart.
(the author doesn’t conclude that U.S. policy is
unjustified, or that Soviet policy is justified), but points (A) The merchants don’t assume that there are no good
out that the conclusion hasn’t been persuasively reasons to restrict advertising, only that restrictions
argued. would also have negative results.
(C) The merchants needn’t examine the objectivity of
9. (C) the government’s report, since their claim is that the
They give us a big verbal clue in this one; “hence” tells report argues against the government’s course of
us that the claim that doctoral dissertations shouldn’t action. If the report were unobjective, we would expect
be required is a conclusion, although at first we can’t it to be slanted in favor of the government position.
be sure that it’s the argument’s main conclusion. (D) is tricky; since the government doesn’t want to
When you look further, you see that the rest of the eliminate advertising, but only reduce its size, you may
passage is indeed intended to establish this main think that the merchants’ argument should make some
conclusion; the argument says what doctoral programs reference to the importance of advertising size.
should do, gives a number of reasons that the However, (D) is too strongly worded; there’s no reason
requirement of a dissertation frustrates these proper the merchants should have to establish a “precise”
goals, and concludes that doctoral disser tations proportion between market-share sizes and amount of
shouldn’t be required in the humanities. As (C) says, advertising.
this is the claim that the whole argument is intended to (E) The question of constitutionality is irrelevant to the
establish. merchants’ argument, which is based not on law, but
(A) No further conclusion is based on the claim about on the restrictions’ effects on business.
doctoral dissertations; instead, the other statements
provide support for that claim.

12
Section II: Logical Reasoning

11. (D) (D) is useless; it points out one similarity between


When tenants of rent-control units have political power, Spanish and English, and tells us nothing about how
and can make or break rent-control laws, they’re language differences relate to perceptions.
always motivated by desire for shor t-term gain. (E) doesn’t tell us whether people who speak
According to the author, short-term gain (smaller rent languages that have no word for gray can perceive gray,
increases) comes from applying rent control. So when so it doesn’t affect the author’s conclusion. Notice the
tenants of rent-control units have the power to enact scope shift: The fact that a color commonly occurs in
rent control, they will also desire to do so. That means nature doesn’t indicate the relative perception of that
they will enact it. What’s the result? According to the color among users of various languages.
stimulus, the long-term result will be a “shortage of
rental units.” Finally, we know that in “many 13. (B)
municipalities” tenants of rent control units do have We’re asked for a choice that plugs the gap in the
this political power. That leads at last to (D); in many argument; that is, a choice that makes the connection
municipalities, there will eventually be a shortage of between vocabulary and perception. That’s (B): If each
rental units (owing to the rent-control laws perpetuated language has a word for every sensory perception that
by those politically powerful tenants). its speakers experience, then the lack of a word for a
(A) A distortion, signaled by the word “impossible.” sensory quality indicates the lack of a perception. If a
Rent-control ordinances are described as “placing language lacks a word for a color, that means its
limits” on rent increases, not making them impossible. speakers can’t distinguish that color.
(B) The stimulus never hints at the conditions under (A) has nothing to do with colors, which is what the
which rent-control ordinances are likely to be repealed. conclusion focuses on. The fact that most languages
(C) Au-contraire; the author specifically says that rent have words for the most basic family relationships
control has “several negative effects for renters.” does nothing to establish a link between words for
colors and the perceptions of colors.
(E) The stimulus provides no evidence about the results
of a shortage of rental units, in the long term or short (C) observes that different languages make different
term. “categor y distinctions” (i.e. distinctions between
colors, relationships, etc.) without establishing that
12. (A) those differences reflect differences in the ability to
perceive the distinctions.
The author concludes that when a language has fewer
words for colors than does English, speakers of that (D) says that the existence of words for cer tain
language must be “perceptually unable” to distinguish categories reflects the importance of the categories for
as many colors as can English speakers. So the author the speakers, but not that the existence of those words
believes there’s not just a linguistic difference, but reflects speakers’ ability to perceive the categories.
there’s an actual difference in perception. (A) (E) gives the example of languages that don’t have
undermines this conclusion by severing the connection words for colors that aren’t encountered by their
between words and perceptions. Russian has two speaker, but fails to show that the speakers of those
different words for two shades of blue; English has only languages are unable to perceive those colors.
one word for both shades, yet English speakers are
able to distinguish between them. This suggests that 14. (C)
the lack of a word to describe a color doesn’t imply lack Cynthia presents a scenario where two individuals
of ability to perceive that color. believe they are morally obligated to take mutually
(B) points to a basic similarity in languages, which contradictory courses of action. According to Zachary,
doesn’t help us decide whether or not differences both individuals have the right and duty to pursue
between languages point to differences in perception. their “morally obligatory” courses of action without
(C) describes how Khmer applies words for colors interference. The artist has the right and duty to
differently from English speakers, but tells us nothing prevent the demonstrator from destroying the
about how the Khmerians perceive the colors in pornographic art; the demonstrator has the right and
question. Therefore, it doesn’t affect the link between duty to destroy the art. At the same time, the artist has
language and perception. no right to interfere with the demonstrator and vice-
versa. Zachary’s principle leads to a contradiction, and
as (C) says, it’s untenable on its own terms.

13
PrepTest 14 Explained

(A) Cynthia doesn’t attack Zachary’s concept of moral didn’t make enough money from these other source to
obligation, but instead attacks his account of the rights offset the paltriness of the revenue from user fees.
and duties that moral obligations impose. (A) is way out in left field; we have no grounds for
(B) No; Cynthia attacks a logical contradiction inherent deducing how far most citizens live from other airports
in Zachary’s notion of the rights and duties imposed by (we don’t even know there are any other airports in the
moral obligations. county).
(D) The problem isn’t that the term “moral obligation” (B) goes outside the scope of the stimulus by
is understood differently by different people, but that discussing the will of private aircraft owners; all we
it’s possible for different people to feel opposite moral know is that they haven’t paid as much in user fees as
obligations. expected, but we have no idea why they haven’t.
(E) The problem isn’t that Zachary’s understanding of (C) involves a major scope shift: The stimulus discusses
moral obligation can’t be applied to artists, but that it the airport’s operating expenses, but tells us nothing
becomes contradictory when it’s applied to people with about the airport’s construction expenses.
opposed obligations. (E) Another left-field choice; we’ve no evidence for
concluding what will happen in the future.
15. (E)
Cynthia presents a case where two people felt moral 17. (C)
obligations that brought them into direct conflict, and The consumer activist complains that deregulating
we’re looking for a choice that also presents an major airlines has worked to the disadvantage of
impossible situation, and that’s (E). The health everyone who doesn’t have easy access to a major
inspector must force the householder to get rid of airport. The industry representative counters that, on
some of her cats; the householder must keep all the the contrary, thanks to regional airlines, there are now
cats she has, and collect more if possible. It’s more regular flights out of most small airports than
impossible for both to fulfill their moral obligations. there used to be. Do you see the problem, in the form
(A) Only one of the two people here actually feels a of the scope shift? The activist is talking about
moral obligation, so we can cross this choice off on “disadvantage” and the representative is talking about
those grounds alone. “number of flights.” We need something to connect
(B) The moral obligations here are in harmony with those ideas, and we find it in (C).
each other; no good. (A) is too strong. The representative doesn’t have to
(C) Scope shift: This deals with opposing beliefs about make such an absolute claim in order to rebut the
a third party’s moral obligations; it doesn’t force the activist’s claim that everyone without access to a
two bankers into inevitable conflict over their own major airport has suffered.
moral obligations in the same way as the characters in (B) sounds like the evidence from the consumer
Cynthia’s argument. activist’s argument (or at least something that’s
(D)’s problem is that we can’t be sure that the moral implied by this evidence), but is not crucial to the
obligations are mutually contradictory; it may be validity of the industry representative’s point.
possible to maximize shareholder profits while (D) The representative refutes the activist’s claim of
designing an energy-efficient building. (D) is missing “disadvantage” by pointing out the increased number
the element of impossibility. of flights. “Cost” is outside the scope, and so certainly
isn’t relied upon in the representative’s argument.
16. (D) (E) is too general; the representative needn’t assume
The author says that the county airport was unable to that an increase in competitors is always a long-term
pay its operating expenses, and blames the fact that it advantage. Moreover, it’s not even clear that there has
didn’t get as much money as it had expected from user been any increase in the total number of airlines.
fees. But the airport was attempting to cover its
expenses only partly by charging user fees, which 18. (D)
implies that money would be coming from some other The consumer activist claims that deregulation worked
source or sources as well. Therefore, if the airport to the disadvantage of everyone without access to a
couldn’t pay its expenses, it must also be true that it major airpor t. Why? Because the removal of
government regulations allowed major airlines to

14
Section II: Logical Reasoning

abandon unprofitable routes “as they promptly did.” 20. (B)


That means, as (D) points out, that the regulations had The spokesperson concludes that, despite the
been to some degree responsible for the fact that the reduction in subsidy, the railroad’s ser vice has
airlines were maintaining their less profitable routes. remained satisfactor y. What’s the evidence? The
Use the denial test; if the regulations weren’t partly number of passengers has increased. The conclusion
responsible, then their removal wouldn’t have caused and the evidence use different terms: “service” and
a wholesale abandonment of the unprofitable routes. “number of passengers.” The assumption must
(A) The activist claims that deregulation created new connect those terms; it must show that the number of
disadvantages for those without access to major passengers reflects the quality of service. (B) does the
airports, but needn’t assume that before deregulation job: the spokesperson is assuming that unsatisfactory
there were no disadvantages for those without such service would cause some people to refuse to travel by
access. train. Otherwise, no matter how bad the service was,
(B) The activist needn’t assume anything about what the number of passengers would be unaffected.
should be done, since he never addresses that (A) Taxpayer wishes play no part in the argument.
concern in his argument; moreover (B)’s claim about (C) Another scope shift: The spokesperson merely
“any sizable group of consumers” is clearly too general concludes that the service has remained satisfactory;
in scope. she needn’t assume the much stronger statement that
(C) is also too general; the activist says that in this the service improved.
case deregulation created disadvantages, but needn’t (D) The spokesperson said that it’s difficult to maintain
assume that regulation “almost always” is advantageous. quality of service when there’s subsidy reduction, not
(E) The activist’s argument never mentioned “regional that it’s impossible. Even so, notwithstanding that
airlines” so you can be pretty sure he’s not assuming difference in degree, this is still just a restatement of
anything about them. Moreover, the argument doesn’t the author’s evidence.
discuss “quality of service” but concentrates on access (E) goes outside the scope by venturing into the future,
to service. which the spokesperson never discusses; moreover,
the spokesperson’s argument needn’t assume anything
19. (E) about revenue produced by the passengers.
The author takes issue with the report’s conclusion
that nitrogen deposited by air pollution is good for 21. (A)
North America’s eastern forests. He points out that The author concludes that the risks of the elective
European scientists have found that when too much surgeries performed before the five-week period were
nitrogen is deposited, trees begin to die. He says that unnecessary. To weaken this conclusion, we’ll need to
this probably applies to North America’s eastern find a choice that makes elective surgery seem more
forests also, which are already suffering from excess necessary, and (A) does just that. If elective surgery
nitrogen. The point is stated in (E): The report’s prevents a condition from worsening into a life-
conclusion is wrong, and the nitrogen in air pollution, threatening, thus necessitating even riskier surgery,
far from benefiting trees in North America’s eastern then the conclusion that the elective surgery was
forests, will probably cause them to die. performed unnecessarily is weakened significantly.
(A) The author disagrees with the report’s conclusion, (B) Whether or not the patients of the elective surgery
so his “main point” can hardly be simply to state that were informed of the risks does nothing to weaken the
conclusion. conclusion that the risks of performing elective surgery
(B) The author never says anything to suggest that the were unnecessary. It’s an irrelevant consideration.
capacity of forests to absorb nitrogen increases as the (C) Irrelevant comparison: The rate of surgery in this
forest becomes nitrate-saturated—quite the opposite! area as opposed to “elsewhere” has no bearing on the
(C) Au-contraire—the author states that “this finding is argument’s validity.
likely to apply to forests everywhere.” (D) The conclusion is that the risk of elective surgeries
(D) makes a false comparison not based on the were incurred unnecessarily. A statement that shows
argument; the author never contrasts eastern North that such surgeries are generally less risky overall than
American forests to other North American forests. emergency surgeries doesn’t show that the risks were
necessary. If anything, it leans in the other direction.

15
PrepTest 14 Explained

Moreover, the reason why elective surgery is generally (A), (B), and (C) all deal with ways to improve hurricane-
less risky than emergency surgery, choice (D), doesn’t stricken areas. Nothing in the passage allows us to
get at the issue of the need for the elective surgeries; argue against any of these. These may or may not be
it’s another irrelevant side issue. good proposals, but we have no way of telling from the
(E) The reason that some patients die (infection) doesn’t information in the passage. Therefore, the passage
weaken the conclusion. This choice doesn’t even specify doesn’t support an argument against these proposals.
which type of surgery we’re talking about, so it’s too (D) First of all, “coastal lands” is way too broad for the
vague to weaken the argument. scope of this stimulus. Secondly, (D) doesn’t go
against the thrust of the passage in any way, so there
22. (D) are no grounds for an argument against this proposal.
Evidence: A painting that is similar to van Gogh’s work
in several ways, but is not in any catalog of his work. 24. (D)
Conclusion: On the sole basis of these similarities, it is The conclusion here is that prohibition actually had the
“virtually certain” that this is a new, never-before opposite of its intended effect: Since the death rate
catalogued van Gogh. The flaw should seem fairly went up during this period, prohibition made people
obvious; the owner doesn’t consider other possible use more alcohol than they would have without the
explanations, such as the most obvious: Another artist ban. To weaken the argument, look for something that
could paint just like van Gogh. In typical wordy LSAT provides an alternative explanation for the increase in
style, that’s essentially what we get in (D). the death rate. If something else was responsible for
(A) The author doesn’t seek “general agreement.” If the death rate increase, then the conclusion that
she did, maybe someone would point out the error of prohibition led to more drinking (an attempt to explain
her ways. the mortality statistics) would be weakened. (D) is the
only choice that doesn’t weaken the argument. How
(B) The author has no logical obligation to substantiate
the alcohol that contributed to the alcohol-related
her evidence regarding the subject matter with an
deaths between 1951 and 1963 is produced, and the
expert authority. We have to find how the path between
fact that it’s imported, are irrelevant to the claim that
evidence and conclusion (i.e., the reasoning) is flawed,
prohibition fosters more drinking.
not ways in which the author failed to prove his or her
evidence. All of the wrong choices sever the connection between
the evidence (fatalities) and the conclusion (prohibition
(C) This choice mistakenly plays off the last few
makes people want to drink more). They do this by
words. That fact that the author believes that the
providing alternative explanations for the fatalities:
painting is “a bargain at its price” does not by itself
imply that the owner is assuming that the only reason (A) shows that the fatalities recorded during the five
to buy art is to make a profit. year period could have been the result of alcohol abuse
before prohibition went into effect.
(E) No; there’s no indication of self-interest at work here.
(B) brings up the possibility that the fatalities may not
23. (E) even be alcohol-related.
This passage discusses government-subsidized insurance (C) If the increase in the death rate during the stated
which makes it feasible for people to buy houses in period is simply part of a larger trend, then the ban on
hurricane zones by allowing them to recoup a high alcohol may have nothing to do with people’s desire to
percentage of losses in the event of a major storm. In drink.
light of this, the government bill that shouldn’t be (E) If this is true, then the death rate may have gone
supported is found in choice (E): Why would anyone up due to a decline in life-saving medical attention for
pay for government-subsidized insurance if there alcohol users, rather than an increase in drinking.
existed a contingency fund that protects the owners of
uninsured houses? An argument against the proposal 25. (E)
in (E) is that its redundant with the existing insurance The editor concludes that the odds are overwhelming
structure and would result in a loss of government (19 to 1) that the letter was written by a man. Why does
revenue. the editor think so? Because fewer than 5% of the
professors at such schools are women. If the editor
had no other information, this might seem to be a

16
Section II: Logical Reasoning

logical conclusion. However, there is another piece of


info that certainly may sway the odds—the doctor’s
name is Shirley!! Taking this into account, the editor’s
reasoning seems flawed; it’s much more likely that the
letter was written by a woman (it’s not 100%, because
theoretically, there could be a man named Shirley,
although it’s not very likely). This flawed reasoning is
mirrored in (E). Since 19 out of 20 animals are
mammals, and fewer than 1 out of 20 are birds, it is
concluded that the odds are overwhelming (95%) that
the animal Emily saw is a mammal. This totally
disregards the fact that the animal was flying, which
sways the prediction heavily in the other direction,
much like the name “Shirley” did in the original.
(A) interprets statistics a little too strictly, but doesn’t
have the element we’re looking for; a factor that sways
the normal odds associated with a prediction.
(B) deals with not having the numbers of the evidence
(15 with 1 in 20 evidence) which is not parallel with the
stimulus.
(C) The hypothetical “if he had graduated,” along with
“a likely result” throw this choice off track.
(D) contains an out and out statistical flaw, but this flaw
doesn’t mirror the one perpetrated by the editor. In the
stimulus and correct answer (E), probabilities are used
to predict the nature of one specific thing, whereas in
(D), the stats are used (erroneously) to formulate a
conclusion about an entire group.

17
PrepTest 14 Explained

SECTION III: • Note, however, that this passage conforms to a


structure that’s common in science passages: a
READING COMPREHENSION scientific process is described and competing
explanations of it are then evaluated.
Passage 1: Earth’s Magnetic Field
The Questions:
Questions 1–6
1. (C)
Topic and Scope: Earth’s magnetic field; specifically,
reversals in the magnetic field’s polarity. This choice is a nice paraphrase of lines 16–21, which
state that the Earth’s magnetic field is generated by
Purpose and Main Idea: The author’s purpose is to the movement of free electrons in the hot metallic fluid
discuss the process that results in reversals of the that constitutes the Earth’s outer core.
magnetic field’s polarity, as well as two distinct (A) distorts the “asteroid-impact hypothesis.” According
hypotheses that try to explain these reversals; since to this hypothesis, “heat circulation in the outer core”
this text is essentially descriptive, there really isn’t a is affected by changes in the polar ice caps, not the
specific main idea, though the author does say (in the other way around.
last paragraph) that the “heat-transfer hypothesis” (B) combines elements of both the “heat-transfer
offers a better explanation than the “asteroid-impact hypothesis” and the “asteroid-impact hypothesis” into
hypothesis.” a statement that has no support in the text.
Paragraph Structure: Paragraph 1 introduces the topic (D) is par t of the “asteroid-impact hypothesis”—a
and scope of the passage. The rest of this paragraph hypothesis that may or may not turn out to be correct.
and all of paragraph 2 provide a lot of scientific (E) also distorts information in the text, which says
factology about the Earth’s magnetic field, particularly that reversals in magnetic field polarity may be related
facts about what is known (not much) about the to changes in “the heat circulation pattern of the outer
process of polarity reversal. core fluid…” (lines 27–28).
Paragraph 3 outlines both the “heat-transfer hypothesis”
and the “asteroid-impact hypothesis” of polarity 2. (E)
reversal. In paragraph 4, on the other hand, the author In lines 59–63, the author’s objection to the “asteroid-
asserts that the “heat-transfer hypothesis” is a more impact hypothesis” is that it depends on an
compelling explanation of polarity reversal. “extraterrestrial intervention,” which he finds less
compelling than explanations that rest on terrestrial (or
The Big Picture: earthly) events. Of the choices, only (E) presents an
• A good grasp of the passage doesn’t mean extraterrestrial explanation—cometary impact—for the
assimilating all of the details (they can be looked extinction of the dinosaurs.
up should this become necessary). Rather, it Choices (A)–(D) each present an “earthly” reason for
means comprehending what the author’s doing in the disappearance of the dinosaurs—the type of
the text—in this case, describing a scientific reason that the author favors when it comes to
process and two hypotheses that purport to explaining earthly events, whether magnetic field
explain it. reversals or species extinction.
• Although this isn’t a ver y difficult science
passage, it’s still not a good place to begin work 3. (A)
on the section. Why? Because the author’s The author mentions hotter and cooler blobs in the
purpose isn’t entirely clear until late in the context of describing the “heat-transfer hypothesis.”
passage. Passages that begin with a mass of This hypothesis suggests that heat circulation patterns
facts—instead of a clear statement of authorial in the outer core are affected by the way in which heat
intent—are generally best left for later in the is vented from the outer core.
section. Moreover, a brief scan of the question
(B) distorts information in the text, which makes it clear
set suggests that it’s not going to be an
that magnetic field reversal is thought to be caused by
especially easy one.
a disturbance in heat circulation patterns in the outer
core (lines 26–28).

18
Section III: Reading Comprehension

(C) is too vague. The author mentions the blobs only in (B) These two hypotheses were formulated to address
the context of laying out a specific hypothesis about the “underlying causes” of magnetic field reversal.
magnetic field reversal. (C) is also beyond the passage’s That they might also address the issue of reversal
scope: the text dwells on the Earth’s interior, not its frequency is simply a by-product of their primary
exterior. function.
(D) focuses on the substance of the detail, rather than (C) is a “half-right, half-wrong” choice. True, no firm
why the author included it in the text. conclusions about either hypothesis have yet been
(E) also distorts information in the text. According to reached; but there’s no information about the extent to
lines 16–18, it is an accepted fact that the magnetic which the two have been explored.
field itself is produced by the movement of free (E) is beyond the scope of the text. We aren’t told
electrons in the outer core’s fluid. precisely when either hypothesis was formulated.
Moreover, both hypotheses have at least some support
4. (B) among geophysicists.
Lines 8–9 say that geological evidence demonstrates
that magnetic field reversals have been occurring with 6. (D)
greater frequency in the recent past. Changes in oceanic circulation patterns aren’t
(A) Lines 1–3 state that “[i]t is a fundamental tenet of mentioned anywhere in the text as possible
geophysics that the Earth’s magnetic field can exist in contributors to magnetic field reversals. The “asteroid-
either of two polarity states…” A fundamental tenet is impact hypothesis” does speak of temperature drops
something that is agreed upon by everyone in the in and redistribution of seawater, but these events are
discipline. Besides, there’s no sense in the text that different from changes in oceanic circulation patterns.
any geophysicists contest this fact. (A) Geophysicists think that change in heat circulation
(C) is beyond the scope of the text. This passage is in outer core fluid is the basic cause of magnetic field
about the “underlying causes” (lines 14–15) of reversals. They part company, however, over what
magnetic field reversals, not about how fast they occur. causes the change in heat circulation.
The only point made about the speed of these (B) and (E) are components of the “asteroid-impact
reversals is that they occur over a period of thousands hypothesis.”
of years. (C) is part of the “heat-transfer hypothesis.”
(D) The “heat-transfer hypothesis” is simply one possible
explanation that geophysicists have come up with to
account for a process that they don’t fully understand.
To say that it has enhanced their knowledge of
magnetic field reversal is an overstatement.
(E) Magnetic field reversal is thought to result from
changes in heat circulation patterns in the outer core,
not from friction along the boundary of inner and outer
cores.

5. (D)
Although this passage discusses just two hypotheses
about magnetic field reversal, we’re told that others
exist. The last sentence of the text explicitly refers to
“theories that depend on extraterrestrial intervention”
and “theories like the first” (i.e., “earthly” theories).
(A) is beyond the scope of the text. We’re not told what
geophysicists in general think about these hypotheses,
let alone whether “they have sharply divided the
scientific community.” All we can infer from the text is
that there is some support for both hypotheses.

19
PrepTest 14 Explained

Passage 2: Deconstruction (D) focuses on a detail in paragraph 3. Moreover, this


choice reflects the author’s opinion of deconstruction,
Questions 7–13 which isn’t necessarily what deconstructionists think of
deconstruction.
Topic and Scope: the literar y philosophy of (E) This choice contradicts the author’s critical attitude
deconstruction; specifically, the connection between toward evaluating literature in light of “borrowed or
its terminology and its methods. adapted” terminology.
Purpose and Main Idea: The author’s purpose is to 8. (E)
demonstrate that links exist between deconstruction’s This choice is a good paraphrase of lines 15–18.
terminology and its methods; the author argues that (A) The author never claims that deconstruction would
deconstructionist terminology provides clear insights have been impossible without the use of these terms.
into deconstruction’s methods. He refers to them simply to highlight an aspect of
deconstructionist philosophy.
Paragraph Structure: Paragraphs 1 and 2 essentially
point out that deconstructionist terminology reflects (B) is an au contraire choice. Lines 12–15 indicate
the philosophy’s methods. The words—along with their that deconstructionists have chosen neologisms for
prior meanings—that deconstructionists have very specific reasons.
“borrowed or adapted from stock” to define their (C) The author never says that deconstruction “contains
philosophy imply certain things about that philosophy. inherent contradictions.” What he says is that
Paragraph 3 is the heart of the text. This paragraph deconstructionists are on the lookout for
discusses the specific ways in which the term contradictions in the work of others.
deconstruction sheds light on deconstructionist (D) is another au contraire choice. Deconstructionists
methods. Basically, this term, which is taken from the are interested enough “in the relationship between
construction industr y, highlights deconstructionist words and their referents” that they’ve developed
efforts to demolish, rather than simply criticize, literary terminology to illustrate this relationship.
works.
9. (C)
The Big Picture:
The author’s belief about innovation in language (lines
• Many of you may have found this a difficult 1–6) is that it consists of giving new meanings to
passage because of its rather abstract nature. existing words. (C) expresses precisely the same
Don’t worry if you don’t understand all of the relationship—existing components “are made to
points made by the author—you’re not going to function in new ways.”
be asked about most of them. The most None of the other choices reflects the author’s
important thing to pick up on in a passage like fundamental idea about “borrowing or adapting”
this one is the author’s critical tone—that’s something that already exists to serve a new end.
what’ll really help you to answer questions.
• Since this passage isn’t easy, a savvy test taker 10. (B)
might well have left it for last. On test day, begin In lines 44–46, the author sets up a contrast between
work on the reading comprehension section with deconstruction and criticism. In lines 46–56, he
a more “concrete” passage. fleshes out this contrast by defining criticism and
showing how it differs from deconstruction.
The Questions: (A) Lines 46–56 do contain an example of sorts—the
7. (A) example of the building. This example, however, is in
the text because it supports the contrast that the
This choice nicely captures the topic, scope, and author makes.
purpose of the passage.
(C), (D), and (E) are beyond the scope of the text. If
(B) These literary terms didn’t pre-date deconstruction; anything, the author makes an argument in lines
rather, deconstructionists turned these words into 46–56; he doesn’t undermine one (C). Nor does he
literary terms. Besides, this is a mere detail in “codify a system” (D) or “dismiss an objection” (E):
paragraph 2; it’s certainly not the text’s main idea. What system? What objection?
(C) also plays on a detail in paragraph 2.

20
Section III: Reading Comprehension

11. (C)
In paragraph 1, the author argues that the old meaning
of words doesn’t disappear when people use these
words in new ways. In paragraph 2, the author provides
an example of this phenomenon by showing that the
word “signify” conjures up a certain idea, even though
deconstructionists don’t have this idea in mind when
they use this word.
(A), (B) paragraph 2 neither “introduces a hypothesis”
(A) nor “qualifies a claim” (B). It simply backs up an
abstract argument made in paragraph 1 with a
concrete example.
(D) distorts a detail in the last sentence of paragraph 2.
(E) Paragraph 3 “presents a contrasting view”—a view
that takes issue with deconstruction.

12. (B)
In paragraph 3, the author’s disapproval of
deconstruction, which he thinks “has no overtones of
skill or wisdom,” is evident. Equally apparent is his
approval of criticism, which is based on “skill and
insight.”
(A) Deconstructionists—not the author—think that it’s
important to “demonstrate false assumptions and
inherent contradictions.”
(C) The author has problems with deconstructionist
philosophy, not with the number of deconstructionists
(or, for that matter, other critics) who may analyze a
work.
(D) is a metaphor for an analytic process favored by
deconstructionists. The author is critical of this analytic
process.
(E) distorts a detail in lines 32–34. The author doesn’t
make any judgements about text structures; he makes
a judgement about differing modes of literary criticism.

13. (D)
This choice both reflects the author’s generally
negative attitude toward deconstruction, and echoes
his comment in lines 28–30.
(A), (B), (C) The author isn’t “guardedly optimistic”
(A) or “enthusiastic” (C) about deconstruction. Nor
does he “endorse” it in any respect (B). Indeed, he’s
critical of deconstruction’s search for authorial bias, as
well as the way it uses words and neologisms.
(E) is too strongly negative in tone. Besides, according
to the author, deconstructionists don’t think of literary
criticism as a “creative act.” Just the opposite; it’s a
repetitive, “mechanical” process.

21
PrepTest 14 Explained

Passage 3: Stolen Art The Questions:

14. (C)
Questions 14–20
This choice neatly encompasses the author’s topic,
Topic and Scope: Stolen art; specifically, legislation scope, and purpose.
intended to protect “good-faith” purchasers of stolen (A) is an au contraire choice. In lines 25–26, the author
art. contends that reclamation suits haven’t been a
problem; therefore, he argues, legislation to “clarify”
Purpose and Main Idea: The author’s purpose is to
the rights of museums isn’t necessary. Moreover, this
argue against such legislation; the author’s specific
choice is too narrow in focus—the author looks at
main idea is that such legislation is not only
more than simply “the legal position of museums…”
unnecessary to protect good-faith purchasers, but is
also unfair to those who seek to reclaim stolen art. (B) is beyond the scope of the text. According to the
text, James Burke, a museum director, supports the
Paragraph Structure: Paragraph 1 describes the new new legislation; but that’s quite different from saying
legislation designed to protect good-faith purchasers of that museum directors in general urged it on the
allegedly stolen art, essentially saying that the burden government. In fact, the passage doesn’t say anything
of proof lies entirely with those who want to recover the about the attitudes of museum directors in general.
art, and restricting the amount of time that they have (D) is also beyond the scope of the text. First, the
to do so. Paragraph 2 discusses the rationale behind passage doesn’t refer to “clashes” between “museum
the legislation, making the point that without such a professionals” and “members of the academic
law any individual or organization that has made a community.” Second, the passage is about a specific
good-faith purchase of art is in jeopardy of losing it. piece of legislation concerning stolen art, not about
The Keyword “however” at the beginning of paragraph arts-related legislation in general.
3 signals that we’re about to get the author’s opinion. (E), too, is beyond the scope. The passage doesn’t
He argues against the legislation on two grounds: (1) discuss any alleged “desire of some governments to
it’s unnecessary because there haven’t been many use legislation and litigation to recover cultural
“reclamation suits” and (2) it’s unfair to those who property.” Moreover, the author’s attitude toward the
seek to recover stolen art because its stipulations recovery of stolen art is such that he would be unlikely
make it virtually impossible for them to press a claim. to speak of legal “abuses” by governments that
In paragraph 4, the author goes on to argue that a actually sought to reclaim art.
different type of legislation is necessary—legislation
that makes it easier for the rightful owners of stolen art 15. (B)
to find and reclaim their property.
Lines 3–5 make it abundantly clear that the “uncertainty”
The Big Picture: referred to in line 2 concerns “the ownership of art,”
specifically legal challenges to the current owners by
• This passage is a pretty good place to begin work those who claim that the art was previously stolen. The
on the reading comprehension section. Why? The rest of the passage takes up this very theme.
subject matter is ver y accessible, and the
author’s voice is plainly evident from line 25 on. (A) is beyond the scope of the text. The passage deals
Moreover, even before you get to line 25, it’s with the legal issue of ownership of allegedly stolen
predictable that the first two paragraphs are art; it doesn’t discuss the moral issues involved in
simply a set up for an authorial counterargument. ownership of “great art.”
• Don’t worry about taking in all of the author’s (C) If anyone has got questions about the origins of art
various reasons (in paragraph 3) for arguing that works, it’s the people who’d like to reclaim them.
the intended legislation is unfair to those who (D) is also beyond the scope of the text. The passage
want to recover stolen property—you can always doesn’t discuss “disputes” between “cultural institutions
look them up if the questions demand it. vying for the opportunity to purchase” a piece of art.
(E) Current art owners aren’t worried about damage or
theft, but rather legal challenges to their ownership.

22
Section III: Reading Comprehension

16. (C) 19. (D)


In the absence of a statute of limitations on the Paragraphs 3 and 4 clearly demonstrate that the author
recovery of art, Burke is afraid that any country might opposes the proposed legislation with a series of
pass a law at any time to the effect that all art works careful arguments. His attitude, in other words, is best
produced within its territory are “cultural property” that described as one of “reasoned opposition.”
rightfully belong to it. Such a legal decision, he goes on (A), (B), (E) These choices wrongly suggest that the
to argue, could involve museums that possess art author supports—with greater or lesser enthusiasm—
works from such a country in “ruinous” court battles. the proposed legislation. As a matter of fact, in
Choice (C) reflects precisely the sort of scenario that paragraph 4, he proposes counterlegislation.
Burke fears.
(C) The author opposes the legislation on the grounds
(A) Burke is worried about art works currently held by that it is unnecessary to protect good-faith purchasers,
museums, not those that they might be prevented from as well as unfair to those who are legitimately trying to
buying in the future. reclaim stolen art. This type of opposition certainly
(B),(E) Burke is concerned about possible national, doesn’t qualify as “fearful apprehension.”
not international, legal developments.
(D) There’s nothing in the passage to indicate that 20. (E)
Burke thinks that private collectors pose a threat to In paragraph 4, the author advocates legislation that
museums. would make it easier for rightful owners of art to
recover stolen property. A law that mandated the
17. (C) collection and distribution of information about art
In paragraph 2, Burke refers to foreign governments as thefts would certainly make it easier for rightful owners
potential adversaries of his nation’s museums. to recover their property.
(A), (D), (E) Burke never even mentions commercial art (A) In lines 35–37, the author contends that museum
dealers (A), private art collectors (D), or other countries’ publications aren’t a solution to publicizing and
museums (E). locating stolen art.
(B) If anything, Burke views law enforcement officials (B) Who should hold onto art that is the subject of
in his own country as potential allies. He does, after litigation isn’t an issue that the author delves into.
all, praise his country’s new legislation. (C) The author favors those who want to reclaim stolen
property, not museums that would like to keep it.
18. (A) (D) The author’s concerned with the reclamation of
This choice is a nice paraphrase of lines 25–26, where stolen art, not with museum-to-museum sales of it.
the author, on the basis of past history, disputes
Burke’s fears (expressed in the previous paragraph)
that widespread reclamation suits could do serious
harm to current art holders.
(B),(E) The author explicitly states that reclamation
suits haven’t yet become a problem. And certainly not
as a result of supportive legislation (B), which isn’t on
the government’s agenda, or a growth in theft (E),
which isn’t even mentioned in the text.
(C) is an au contraire choice. The author says that
reclamation suits haven’t been a problem up to this
point in time.
(D) How could the author believe that lawsuits are a
sign of frustration, legitimate or otherwise, when he
suggests that they’ve been a ver y infrequent
occurrence?

23
PrepTest 14 Explained

Passage 4: Russian Serfdom, US Slavery (B) contradicts the passage. Kolchin has now written
such a study—and the implication is that it’s at least
Questions 21–27 “adequate.”
(C) refers to a detail in paragraph 3. It entirely leaves
Topic and Scope: Kolchin’s comparative study of out the companion issue of slave resistance in the US.
serfdom in Russia and slavery in the United States;
(D) distorts the passage, which never suggests that
specifically, Kolchin’s identification of impor tant
Kolchin is skeptical of comparative studies. On the
differences between the two systems.
contrary—he’s written one!
Purpose and Main Idea: Author wants to outline (E) De Toqueville is a passing detail in paragraph 1.
Kochin’s findings, “especially with regard to the
different kinds of rebellion exhibited by slaves and 22. (B)
serfs.” (B) paraphrases the first 2–3 sentences. It’s as simple
as that.
Paragraph Structure: Paragraph 1 explains that very
few historians have compared slavery and serfdom, (A) contradicts the opening sentence: “Until recently,
with the final sentence introducing Kolchin’s book. few historians were interested in analyzing the
Paragraph 2 notes key differences in the number and similarities and differences between” serfdom and
population size of slave- or ser f-owing estates in slavery.
Russia and the US. Paragraph 3 explains that these (C) distorts the passage, which never suggests any
demographic differences “partly explain differences in “inability” on de Toqueville’s part—he simply never
the kinds of resistance that slaves and serfs practiced addressed the issue of abolition in the two countries.
in their respective countries.” Resistance was common (D) Like (A), (D) contradicts the first sentence.
in both countries, but large, organized, armed
(E) Half-right, half-wrong. De Toqueville “recognized the
rebellions were more common in Russia. Conflicts
significant comparability of the two nations…” but his
between US owners and slaves were frequent but less
commonality with other historians was that he never
collective, mostly because there were fewer workers on
compared slavery and serfdom.
each estate in the US.

The Big Picture: 23. (C)


• This history passage focuses on a book. That’s (C) is consistent with key info in paragraphs 2–3.
common. What’s unusual is that the author gives Organized rebellions were common in Russia because
no direct critical assessment of Kolchin’s ideas. most serfs lived on large estates, which encouraged
The author just lays out Kolchin’s findings, collective forms of resistance. A parallel idea would be
implying that his work is satisfactory. that any organized rebellions occurring in the US would,
like those in Russia, have occurred on large estates.
• The academic lingo is long-winded, but the
paragraphs organize things nicely—certainly a (A) would weaken Kolchin’s theme. His idea is that
dividend for the pressured test taker. Paragraph 1: Russian conditions tended to encourage hugely
a long intro to Kolchin; paragraph 2: the key explosive revolts, not merely “smaller collective acts of
demographics; paragraph 3: differences in forms defiance”—the volnenie.
of rebellion. (B) picks up on info in the second sentence of paragraph
3: Much of the rebelliousness in both countries—which
The Questions: took the form of silent sabotage—escaped the
historical record. Kolchin’s major point, however, is
21. (A) that demographics accounted for differences in the
(A) captures the gist of the key final sentence of occurrence of over t armed rebellion, not silent
paragraph 1, and it echoes the thrust of paragraph 3. sabotage. Plus, if there were armed revolts in the US
Plus it echoes the content of paragraph 2, which that escaped the historical record, then that would raise
supplies the demographic facts that explain the questions about, not strengthen, Kolchin’s findings.(D)
differences in forms of slave and serf resistance. goes on at length without saying anything especially
relevant.

24
Section III: Reading Comprehension

(E) would undercut Kolchin, who points to a revealing


correlation between the likelihood of serf revolt and the
fact that most Russian estates were managed by
intermediaries.

24. (C)
(C) paraphrases the passage. The author’s surprised
that the coincidence of abolition in Russia and the US
“failed to arouse the interest of scholars.” As (C) says,
that coincidence should have prompted comparative
study of the two institutions.
(A) The passage never explains what led to abolition in
either country.
(B) The passage never suggests that de Toqueville
missed something he should have noticed.
(D) No criticism of Kolchin’s book is ever made.
(E) is inconsistent with the passage, which stresses
key differences between slavery and serfdom.

25. (A)
This choice repeats info in the last sentence of
paragraph 1: “…historians might have been put off by
the forbidding political differences between nineteenth-
century Russia and the United States.” None of the
other choices are suggested.

26. (D)
(D) is a simple paraphrase of the point made in the
last sentence of paragraph 2: “In Russia most serfs
rarely saw their owners…”
(A) Never suggested. The only action attributed to the
nobles was their relying on intermediaries to manage
their estates.
(B) Ditto. Never suggested.
(C) Never suggested.
(E) Never suggested.

27. (D)
(D) is pretty directly stated in the last sentence of
paragraph 2, the same material that provided the
answer to question 26.
(A) The passage never says that any estate owner—
Russian or US—was “prepared for collective protest.”
(B) Au contraire. Most southern planters—98% of them,
implies paragraph 2—owned fewer than 100 slaves.
(C) Tempting, since most Russian estates were
managed by intermediaries, but not clearly implied. (D)
is far better.
(E) Au contraire, according to paragraph 2.

25
PrepTest 14 Explained

SECTION IV: motive for not staying in touch with Enrique—he’s


simply an airhead (C).
LOGICAL REASONING
(B) would work if it told us that Michael’s motives for
donating to charity were selfish, but instead it tells us
1. (E) that his motives for telling Sarah he donates to charity
We’re asked to explain why measuring the cholesterol are selfish.
levels of their blood apparently enabled participants
who were given kits to lower their cholesterol levels 3. (C)
more than those who weren’t given kits. As (E) has it, We’re asked to explain why, after widespread publicity
people who had kits and could therefore measure their about a study that showed that bottled water is
levels were more motivated to lower these levels. In generally less safe to drink than tap water, sales of
other words, they worked harder at it than the bottled water continued to go up. The key phrase here
participants who didn’t get kits. is “in many cases.” This implies that not all the bottled
(A) is too vague. It implies that the readings of the water is worse than tap water, which allows the
people who got kits were inaccurate, but that could possibility of choice (C): While bottled water overall
mean that the people with kits actually lowered their may not be safe, if some popular brands of it are even
levels by even more than 15 percent, which would only safer than tap water, then these brands could account
worsen the discrepancy, so (A) is no help. for the increase in sales.
(B) rules out a possible alternative explanation. If (A) provides useless detail about why bottled water is
participants with the kits ate lots of foods that lower less safe than drinking water, but we want to know why,
cholesterol levels, that might explain why their levels given that this is so, people are buying more bottled
were lower, but (B) says they avoided such foods, water than ever.
which only makes things even more mysterious. (B) and (E) rule out possible alternative explanations.
(C) is irrelevant. How could using the kit more frequently In (B), if consumers preferred the taste of bottled
during the first two months explain why people with kits water to tap water, that might explain why they continue
ended up with lower cholesterol levels? drinking it when they know it isn’t safe, but (B) says
(D) provides useless background information. So what they can’t distinguish between the two, so it’s no help.
if everyone lowered their cholesterol levels somewhat, In (E), if frequent government warnings caused
or if this was achieved in the first three months? That consumers to ignore the study, that too could explain
doesn’t resolve the discrepancy between the cholesterol the increased sales, but (E) instead says that frequent
levels of those with kits and those without ’em. warnings make consumers even more leery of unsafe
foods, which only reinforces the paradox.
2. (D) (D) implies that the study has caused increases in
To match the principle given, we need a situation in bottled-water sales to slow somewhat. So what? The
which someone either doesn’t praise a seemingly point is that sales continue to increase, and (D)
generous act because he or she believes it was done doesn’t explain why this is so.
for selfish reasons, or doesn’t condemn an apparently
selfish act because he or she believes it was not done 4. (C)
for selfish reasons. We get the former in (D): Margaret Evidence: There are nonrenewable, economically useful
offered to share her house with the French family not raw materials on Earth. Conclusion: If these materials
because she wanted to be generous but because she can’t be found somewhere else besides Earth, people
wanted to use their apartment—a clearly selfish will no longer be able to achieve what they now achieve
motive, so Daniel was correct not to praise her. using the materials. You may have been able to pre-
(A), (C), (E) All of these choices fail one the same phrase an answer to this one: Maybe they can use
basic count: The principle involves withholding praise other materials that are renewable to accomplish what
or condemnation, and the correct choice must adhere is currently accomplished with the original materials.
to this element. The principle also concerns people’s The author has assumed this is not an option, and (C),
motives in performing certain actions, but we don’t by breaking down this assumption, undermines the
even know Monica’s motives for offering to help argument.
Caroline (A), and William doesn’t have any apparent

26
Section IV: Logical Reasoning

(A) is beyond the scope. Evidence and conclusion 6. (A)


deal with economically useful resources that are The big assumption in this argument is that we can
nonrenewable; renewable resources are irrelevant. gauge the level of heroin use solely by the number of
(B) is irrelevant. The author never claims that it’ll be number of hospital emergency room visits by heroin
easy to find raw materials in outer space, so denying users. But notice that we’re not even told that these
that it’s easy doesn’t affect her reasoning. heroin users showed up in the emergency room for
(D) involves a scope shift. The issue is whether or not heroin-related incidents. The number of heroin users
people will be able to accomplish what they now that showed up in emergency rooms in the 80s could
accomplish with certain resources, not whether or not have increased while the use of heroin didn’t if more
the things they accomplish are worth accomplishing. heroin users went to the emergency room for other
reasons than they did in the past. (A)’s increase in the
(E) provides useless background information. The author
use of automatic weapons and thus in the risk of injury
specifically uses the word “eventually,” so who cares
to heroin users makes it possible that there were more
how long it will take for these nonrenewable resources
emergency room visits because of increased gunshot
to be depleted? A few hundred years, or a few
wounds among heroin users, not because of any
thousand years, it makes no difference—we’re
increase in heroin usage.
interested in whether or not people will be able to
accomplish what they now accomplish using those (B) wouldn’t account for the author’s evidence. If
resources, and (E) doesn’t address this. users are less likely to get infected by smoking this
new form of heroin, then it would seem there should
5. (B) have been fewer, not more, emergency room visits by
heroin users.
In the first experiment, levels of naturally occurring
salicylic acid in the resistant plants went way up when (C) doesn’t satisfy the first requirement in the stem:
those plants were infected with tobacco mosaic virus; The ability to accommodate the increase in heroin
this didn’t happen in the nonresistant plants. In the users does nothing to explain why that increase
second experiment, the nonresistant plants that were happened in the first place.
injected with salicylic acid didn’t become infected when (D)’s the au contraire choice; it claims that heroin use
exposed to the virus, while the ones that didn’t get did increase. But we’re looking for a scenario where
injections fell victim to the disease. We definitely have emergency room visits went up while heroin use didn’t,
a pattern developing here. It appears that salicylic acid so (D)’s no good.
production has something to do with the way that (E) provides useless background information. Knowing
tobacco plants protect themselves from the disease what types of heroin-related ailments caused heroin
caused by tobacco mosaic virus. users to visit emergency rooms is no help in our effort
(A) Scope shift: Neither experiment dealt with curing to show that emergency room visits went up while
plants that were already diseased, so we can’t infer heroin use didn’t.
that injections of salicylic acid can accomplish this.
(C) Read carefully! In the first experiment, levels of 7. (C)
naturally occurring salicylic acid didn’t increase in the As mentioned above, the author assumes that there’s
nonresistant plants infected with the virus, implying that, a connection between the number of emergency room
contrary to (C), these plants do produce some salicylic visits by heroin users and the incidence of heroin
acid, just not as much as the resistant plants do. usage. To use the Denial Test, if there were no
(D) is beyond the scope. The stimulus never discusses correlation between the two, the author’s argument
how to test whether an uninfected tobacco plant is would make no sense, so (C) must be assumed.
resistant to the mosaic virus, so we have no idea how (A) is beyond the scope. The author deals with
this would be done. emergency room visits by heroin users; nothing is
(E) too is beyond the scope. We don’t know if it’s assumed about when or why heroin users seek medical
possible to increase salicylic acid production in care. To use the Denial Test again, it wouldn’t affect
nonresistant tobacco plants, since the stimulus never the argument at all if, contrary to (A), heroin users
dealt with this issue. All we know is that injecting these typically seek medical care in the early stages of
plants with salicylic acid helped fight off the virus. addiction.

27
PrepTest 14 Explained

(B), since it would weaken the argument, can’t be (next winter). This hypothetical future event (“if prices
assumed. If heroin users visit emergency rooms are high next winter”) plays no part in the original.
repeatedly, then it’s possible that the number of (D) correlates a diet high in vegetables with good
emergency room visits has gone up because of repeat health, but rather than argue, as the stimulus and (E)
visits and not because of any increase in heroin use. do, that the first causes the second, (D) instead
(D) is reminiscent of choice (B) in Question 6. If the concludes with a recommendation: everyone should
new methods of using heroin are less dangerous, eat vegetables. No recommendations are found in the
shouldn’t the number emergency room visits have stimulus, so (D)’s not parallel.
declined rather than increased? (D) makes no sense,
so it can’t be assumed. 9. (D)
(E) It’s not necessary to the argument that the heroin A bit of disguised formal logic here: In order to be
users who go to emergency rooms identify themselves elected, one cannot support the new plan. Everyone
as such, just that the hospital is aware that they are that understands economics doesn’t support the tax
heroin users (otherwise, there’d be no way to tell that plan. Therefore, only someone who understands
the number of visits by heroin users has increased). To economics can be elected. You should be able to pre-
use the Denial Test again, it wouldn’t affect the phrase the problem with this reasoning: People who
argument if heroin users did not identify themselves as understand economics aren’t necessarily the only
such when they visited emergency rooms. people who don’t support the new tax plan. There
could easily be people who don’t support the plan,
8. (E) don’t understand economics, and yet can still be
In the stimulus, we get a correlation between two elected.
phenomena (sunspot activity and popular uprisings); (A) isn’t a possibility that’s overlooked; in fact, it’s
then a connection linking the two: Sunspot activity the crux of the second premise in the argument.
causes more positive ions, which cause greater anxiety (B) There could be people who truly understand
and irritability. The argument then concludes with an economics yet have no chance of being elected due to
assertion of causation: Sunspot activity must be at other reasons; this doesn’t undermine the conclusion
least partly responsible for popular uprisings. So we that understanding economics is necessary to be
can abstract the logical thrust of the argument: elected. So even though the author doesn’t
Correlation between two things; a supposed acknowledge the group in (B), the existence of this
connection; therefore, causality. We get this in (E): group doesn’t show why the argument is flawed.
correlation (between offices with lots of windows and
(C) Same as (B): In order to be elected, one can’t
high productivity); the connection (more exposure to
suppor t the tax plan. Non-suppor t is therefore
natural light increases alertness); therefore, causation
necessary to being elected. That doesn’t mean that
(the greater number of windows is at least partly
it’s sufficient—nothing requires that ever y non-
responsible for certain people’s higher productivity).
supporter is eligible for election. Again, one could be
(A), unlike the stimulus, doesn’t argue that the against the tax plan and still fall short for other
correlation between the birds’ flight patterns and reasons. So, like (B), the possibility that this group
certain events shows that the first caused the second; exists doesn’t show why the argument is flawed.
instead, it claims that the flight patterns were the
(E) Since (E)’s claim is about people who aren’t
result of some other factor that “also” helped cause
electable, it has no bearing on the author’s conclusion.
the events.
We’ve already established in the other answer
(B) argues for a correlation between Ridgeview’s choices that there can be a whole host of reasons that
academic worth and the success of its graduates, but people aren’t electable other than the lack of an
it doesn’t contain any kind of long causal chain; understanding of economics.
moreover, it’s completely unlike the stimulus argument
in that it argues against another claim: Ridgeview’s 10. (A)
poor academic reputation.
In responding to the interviewer’s objection that some
(C)’s correlation-to-causation argument is similar to insomniacs don’t respond to treatment, the therapist
that in the stimulus, but it also contains another claims that their failure to respond merely proves that
element—an argument from past (last winter) to future these people didn’t rigorously adhere to their
treatment. The problem with this reply is that it doesn’t

28
Section IV: Logical Reasoning

allow for the therapist to be wrong; no matter what argues that socialists analyze history in order to work
evidence anyone produces that some insomniacs to transform capitalist institutions and thereby bring
failed to respond to the therapist’s treatment, the about socialism. And the conservative claims history
therapist can simply argue that these people didn’t try occurs through “individual struggle,” among other
hard enough. In other words, the therapist is always things, so apparently, he too thinks people can affect
right, or, as (A) rather pompously puts it, his reply the course of history.
“precludes the possibility of disconfirming evidence.”
(B) There’s no ambiguity about the term “treatment”; 12. (E)
the therapist uses it to refer to the same thing as the To recap, the conservative’s view of history is that it
interviewer does: treatment for insomnia. occurs through accident, contingency, and individual
(C) is irrelevant. Neither the interviewer nor the therapist struggle. The socialist, meanwhile, believes that
discusses the reasons for patients’ insomnia; they’re people can work to bring about socialism by
interested in cures, plain and simple. transforming the institutions of capitalist society. So
far, no disagreement—both hold that individuals can
(D) There’s no need for statistics because the therapist
affect the course of history. But if, as (E) has it, the
doesn’t argue on the basis of statistics. Rather, he
conservative also thinks that history is mostly the
makes the sweeping claim that anyone who doesn’t
result of accident and that it’s therefore impossible for
respond to his treatment just isn’t trying hard enough.
people to bring about large-scale social changes, then
(E) is beyond the scope. Insomniacs who would improve he must disagree with the socialist’s statements.
on their own, without treatment, aren’t part of the
(A) is beyond the scope. Neither the socialist nor the
argument. The argument concerns insomniacs who
conservative discusses the possibility of predicting
don’t respond to any treatment, not those who do
present forms of capitalism, so this is irrelevant.
respond to a lack of treatment.
(B) This is precisely what the conservative argues:
11. (E) that history only appears inevitable in retrospect. But
since the socialist doesn’t think history is inevitable
The conservative argues that history is not inevitable,
either, they don’t disagree on this point.
despite the fact that socialist arguments begin with an
analysis of history and claim to locate trends therein (C) is beyond the scope. Neither radical changes in
that lead ineluctably to socialism. The socialist replies social structures nor the results of those changes are
that if socialists thought history was inevitable, they mentioned, so we don’t know the socialist’s or the
wouldn’t attempt to transform capitalist institutions, conservative’s opinions about them.
and that socialists analyze histor y in order to (D)’s also beyond the scope. The socialist never
understand these institutions and thereby transform discusses the role of accident or contingency in
them. Thus the source of the dispute is why socialists bringing about socialism, so it’s possible she would
analyze history: to argue that socialism is inevitable, agree with the conservative’s belief that socialism
as the conser vative believes, or to understand arises purely from individual struggle.
capitalist institutions in order to transform them, as
the socialist claims. 13. (C)
(A) Both the socialist and the conservative agree that The author uses evidence that dependence on and
socialism is not inevitable, so this can’t be the source abuse of a drug needn’t go hand in hand in order to
of their disagreement. conclude that the definition of “addiction” must be
(B) is beyond the scope of the dispute. The socialist incorrect. In so arguing, she assumes that the cancer
evidently thinks capitalist institutions need patients who, she claims, depend on but do not abuse
transforming, but the conservative never discusses morphine, are actually addicted to morphine. This
them, so we don’t know whether he disagrees with the assumption is so simple you might have missed it, but
socialist on this point. look at it this way: If those cancer patients were not
addicted to morphine, the author couldn’t use them as
(C) Again, as the socialist points out, socialists don’t
evidence that the definition of “addiction” is incorrect.
think socialism is inevitable, so this can’t be the
source of the disagreement. (A) is tricky, but it’s not assumed. Strictly speaking, it’s
possible that, contrary to (A), the cancer patients who
(D) is a point both agree on. The socialist obviously
depend on morphine do occasionally abuse it—that
believes people can affect the course of history—she

29
PrepTest 14 Explained

wouldn’t affect the author’s claim that dependence and (E) claims that George must have voted to make the
abuse don’t “always” go hand in hand. city hall a landmark, when we’re not even told the
(B) No; the author states that cancer patients can society’s vote on that issue. The original, and the
become addicted to morphine, and then goes on to use correct choice, involve an individual following suit from
this case to show that this doesn’t always lead to a group’s action, but following along the same lines.
abuse. Nothing is assumed about whether cancer Here, city hall is an entirely different issue from the
patients “often” become dependent on morphine; even bank.
if it’s a rare occurrence, that wouldn’t affect the stream
of logic. 15. (C)
(D) is a misreading. The author argues that cancer The author reasons that because the commissioner’s
patients can be dependent on morphine without abusing new proposals are identical to the ones issued by
it; there’s no assumption about cancer patients who do Tsarque, Inc., which has engaged in heavy polluting,
abuse drugs, since they’re not part of her argument. they must therefore be worthless. As (C) points out,
this amounts to dismissing the proposals because of
(E) As Descartes would declare, “Au contraire!” If the
their source rather than their content. Indeed, the fact
author assumed that cancer patients can’t depend on
that the proposals may have come from Tsarque is
morphine without abusing it, that would destroy her
irrelevant; they could still be worthwhile, despite
argument that abuse and dependency don’t necessarily
Tsarque’s environmental record.
go together.
(A) The argument doesn’t assume this “without any
14. (D) justification”; as it points out, Tsarque’s chief and the
commissioner are good friends.
The argument is basically one from whole to part, with
a past-to-present element: Since the panel that (B) The author doesn’t give any version of the proposals,
resolved the Amlec dispute was reasonable and fair, let alone a distorted one.
the argument goes, Judge Khalid, who was on that (D) Although the author rather sarcastically suggests
panel, is reasonable and fair as well. Similarly, (D) that the commissioner couldn’t have come up with
erroneously asserts that because a certain real estate “fresh thinking” on the environment, the argument
company sold fewer houses last year than it had the doesn’t appeal to emotion, as (D) implies; it rejects
year before, Ula Borg, an agent employed by that the proposals because of where they’re from rather
company, must have sold fewer houses last year than than what they say.
she had the previous year. Both arguments attribute a (E) There’s no appeal to authority here; the author
characteristic of a group to a member of that group, mentions Tsarque’s chief solely in order to claim that
and both contain a similar time element—the stimulus Tsarque is the ultimate source of the proposals.
moves from past to present; (D) goes from two years
ago to last year. 16. (A)
(A)’s flawed, but not in the same way as the stimulus: The argument concludes (in the first sentence) that the
It asserts that since, by definition, only an elected definition of “natural foods” should include all plants,
member of the school board can ser ve as its including plants grown with the application of synthetic
representative, and since Marcia Barthes represented chemicals. To justify this conclusion, the author
the board before the principal, she must therefore have redefines the term “natural” as “part of nature.”
been recently elected to the board—not necessarily so.
(B) The argument recommends no specific action (except
(B) The main hook in the original is that an individual a redefining of terms), and in any case, no evidence is
was part of a group in which something happened, and provided to show why this redefinition of terms is
therefore the same will hold true for the individual. In beneficial.
(B), Candalf, who’s decided to become a pediatrician,
(C) is a distortion. The argument uses some scientific
is yet to be part of the larger group.
language, but the key here is redefining the word
(C) Here, the erroneous conclusion is that Diaz must “natural”; the author doesn’t directly appeal to the
be part of the larger group. In the original, however, the authority of scientific methods for this purpose.
individual is already a member of a larger group and
(D) The argument never questions the use of the term
the erroneous conclusion is that what holds for the
“organic.” It’s “natural” that concerns us.
group must necessarily hold for the individual.

30
Section IV: Logical Reasoning

(E) The “position being rejected” is that “organic” (B) makes a useless distinction. If the safety
foods are the only natural foods. Since no evidence is precautions would help some passengers, then the
presented supporting this position, the author can’t recommendation is worthwhile, but only if it were true
possibly be “reinterpreting evidence presented as that pilot-whistling is a reliable predictor of minor
supporting…” accidents.
(C) In this context, it’s not necessary to explicitly
17. (E) define “relatively minor”; this is simply a condition which
There are two courses that Angela needs to complete limits the scope of the argument. It doesn’t affect the
her psychology degree: experimental design and basic logical structure.
developmental psychology. Experimental design must (E) distorts the issue, and entirely omits the major
be completed before taking developmental psych. “On element, whistling. Whether the percentage of small
completing both,” she’ll have earned her degree. So airplane flights that involve relatively minor accidents is
we can conclude that once Angela completes ten or seventy, the basic logic of the argument stays
developmental psych, she will have also completed the same.
experimental design, and will therefore have earned
her degree. 19. (A)
(A) is contradicted by the argument. If developmental You may not have immediately spotted this
psych isn’t offered until next term, and Angela could assumption, but it checks out nicely with the Denial
still complete her degree in two terms, then this course Test. Since the permits govern the discharge of
must only take one term. individual chemicals only, if relatively harmless
(B) is beyond the scope. We have no information on chemicals (discharged under permit) do interact with
which of the two courses is easier. each other in water to form harmful compounds, then
(C) could be true, but it’s also possible that Angela the waterways would be endangered.
has already completed the necessary prerequisites for (B) Since the amount of water flowing through the
the experimental design course. waterway has already been taken into account in
(D) is a scope shift. Our information is all about Angela; determining the amount of chemicals that may be
we have no definite information regarding the general discharged (second sentence), it’s not necessary to
university or departmental requirements. make any assumptions about how swiftly the water
flows.
18. (D) (C) The argument’s conclusion is that the waterways
In 75% of minor accidents involving small airplanes, are protected from harmful effects of chemicals
voice-recorder tapes record the whistling of the pilot discharged under permit. Whether there are chemicals
shortly before the accident. Thus, concludes the which may not be discharged into the waterway under
stimulus, pilot-whistling is a fairly reliable indicator of any circumstances is beyond the scope.
an impending accident. The assumption being made is (D) The argument claims that the maximum amount of
that a whistling pilot is not a normal condition on small chemical discharged as determined by the permits is
planes. After all, if 90% of small airplane pilots whistle not harmful to the waterway, so it’s not necessary to
from take-off to landing, it wouldn’t be much of a assume that permit-holders sometimes discharge less
revelation that most of them are whistling shortly than they are allowed.
before minor accidents. Choice (D) highlights this (E) is beyond the scope of the argument. The question
assumption, exposing the major weakness in the is whether the permit system protects the waterway
argument. from adverse effects, as determined by the permit
(A) Once again, as we’ve seen earlier on this test, system. How those adverse effects are defined is a
we’re not trying to debate the validity of the evidence, separate question.
but rather the reasoning that connects the evidence
with the conclusion. In other words, it’s not the 20. (E)
reliability of the statistics used in the evidence that’s The key here is that Monroe concludes that it is solely
important, but rather how that evidence is used to due to Tip-Top’s hot peppers that he became ill.
formulate a conclusion. Perhaps there is some other factor he has overlooked,
such as the fact that “despite his generally poor

31
PrepTest 14 Explained

appetite” he ate 1) an extra-large pizza, 2) all-you-can- on the waiting list for the other five companies, then
eat shrimp, and 3) two meatball sandwiches. The the number of unemployed trained pilots could, in fact,
existence of such a plausible alternate explanation for be lower than the projected demand, seriously
his illness seriously weakens Monroe’s conclusion. undermining the author’s conclusion.
(A) Monroe’s conclusion is limited in scope only to the (B) contains a scope shift. Even if pilot training will be
three meals he actually ate at the Tip-Top, so it’s necessary in the long run, that doesn’t affect the
logically valid for him to use only those three meals as conclusion, which is about the foreseeable future.
evidence. (C) doesn’t weaken the argument, because we have
(B) is contradicted by the information in the passage; no reason (i.e. no evidence in the passage) to believe
we’re specifically told that after each meal (the that an age imbalance in the pilot work force will cause
presumed cause), Monroe became ill (the presumed a personnel shortage in the foreseeable future.
effect). (D) is an au contraire choice. If the argument takes into
(C) requires us to assume that Monroe does in fact account the current upswing in the aviation industry,
desire to continue to eat at the Tip-Top. that would only strengthen the evidence on which the
(D) contains a scope shift. Monroe’s logic never conclusion is based.
claims that Tip-Top’s hot peppers make everybody ill, (E) is irrelevant background information. The policies
only that they make him ill. of other companies with regard to pilot-training have no
bearing on the industry need for trained pilots as
21. (B) outlined in the argument.
This is similar to a logic games question; we’re asked
to consider the if-clause to be a statement of fact, and 23. (C)
then draw a conclusion based on this new information. The author concludes that people with car alarms
Reduced to formal logic, Monroe’s evidence and should deactivate them when parking in crowded city
conclusion read thus: “If Monroe eats Tip-Top’s hot neighborhoods at night, on the grounds that “whatever
peppers, he becomes ill.” So we can logically conclude the cause (presumably, including the possibility that a
that if Monroe had eaten the chicken with hot peppers, car is actually being stolen), the sleep of many people
he would have become ill. in the neighborhood is disturbed.” For this conclusion
(A) and (C) can both be reduced in formal logic to “If to be valid, the author must assume that allowing
Monroe does not eat Tip-Top’s hot peppers, he will not those people to sleep soundly is more important than
become ill.” Not only is this logically invalid (the fallacy preventing car theft.
of the converse), but it goes against common sense: (A) is an au contraire choice. If the author placed
The fact that hot peppers will make him ill doesn’t neighborhood security over inconvenience, her argument
allow us to conclude that a lack of hot peppers will would fall apart.
ensure that he won’t be ill—Monroe could become ill (B) is tempting, but it’s not necessary for the author to
next time for any number of other reasons. assume that most, or even many times car alarms go
(D) and (E) answer the wrong question. Each provides off that they are false alarms. Her recommendation is
(limited) support for Monroe’s argument. Neither, based on “consideration for others,” and “whatever the
however, is a conclusion which can be drawn from “If cause,” the sound of car alarms wakes people up
Monroe eats Tip-Top’s hot peppers, he becomes ill.” inconsiderately.
(D) The author might believe this, but it’s not a
22. (A) necessary assumption. The author could believe that
The company will not be training more pilots because car alarm owners are considerate in every other
there will be no shortage of trained pilots in the respect; it wouldn’t affect the logic of her argument.
foreseeable future. The evidence: Each of the six major (E) is a scope shift. The author’s recommendation deals
companies (the author’s and the other five) have only with the night-time use of car alarms.
roughly 400 trained pilots on their waiting lists, and the
projected requirement for pilots is only 100 per
company. In other words, there are apparently 2400
trained pilots waiting for 600 future jobs. If, however,
most of the trained pilots on one waiting list are also

32
Section IV: Logical Reasoning

24. (C)
The passage suggests two possible explanations for
lines found in the Peruvian desert: 1) they were landing
strips for alien spaceships, and 2) they were Inca
roads. The investigator supports the first explanation
by rhetorically questioning the plausibility of the
second.
(A) No direct counterevidence to the alien spaceship
theory is mentioned.
(B) The investigator presents no evidence to counter
the Inca road interpretation; he simply questions its
plausibility.
(D) The investigator doesn’t challenge the methods of
those who support the Inca road interpretation; in
fact, he doesn’t mention the developers of this
interpretation at all.
(E) No attempt is made to reconcile the alien-spaceship
interpretation with the Inca road interpretation.

25. (B)
We’re asked to provide support to an interpretation of
the lines as referring to astronomical phenomena. So
we need a choice that suggests a plausible explanation
for both the straight lines and the bird figure to have
some astronomical referent. Choice (B) fills the bill.
(A) and (D) both refer to Native American interest in
astronomical phenomena, but neither mentions the
patterns found in the Peruvian desert, so neither can
be the correct choice.
(C) addresses only the straight line pattern. It doesn’t
mention the bird figure or astronomical phenomena.
(E) makes a useless distinction; the relative ages of
the two desert patterns neither supports nor weakens
the claim that they both refer to astronomical
phenomena.

33
1-800-KAP-TEST | kaptest.com

ÖLL3109A…ä
LL3109A

*LSAT is a registered trademark of the Law School Admission Council. Printed in USA ©2008 Kaplan, Inc.

You might also like